1answer.
Ask question
Login Signup
Ask question
All categories
  • English
  • Mathematics
  • Social Studies
  • Business
  • History
  • Health
  • Geography
  • Biology
  • Physics
  • Chemistry
  • Computers and Technology
  • Arts
  • World Languages
  • Spanish
  • French
  • German
  • Advanced Placement (AP)
  • SAT
  • Medicine
  • Law
  • Engineering
kipiarov [429]
3 years ago
12

I recently (yesterday) took the AMC 10A, can anyone help me with this problem?

Mathematics
1 answer:
lianna [129]3 years ago
8 0
First, we claim that \frac{10^n-1}{9} is the n-digit number with all digits equal to one.

Note that 10^n is a one followed by n zeroes, so subtracting one gives n nines.  Divide that number by nine and you get n ones, completing the proof.

Therefore, we have that A_n = a \frac{10^n - 1}{9}, B_n = b \frac{10^n-1}{9}, and C_n = c \frac{10^{2n} - 1}{9}.

Let x = 10^n.  Then, we have:

c \frac{x^2-1}{9} - b \frac{x-1}{9} = (a \frac{10^n - 1}{9})^2 = a^2 \frac{x^2 - 2x + 1}{81}.

Multiplying by 81 gives:

9c(x^2-1) - 9b(x-1) = a^2 (x^2-2x+1)

Now, note that x=1 is not a valid input, since 10^n = 1 requires n=0, so we safely divide by x-1 to get:

9c(x+1) - 9b = a^2(x-1)

9cx + 9c - 9b = a^2x - a^2

x(9c-a^2) = 9b-9c-a^2

Because this is now a linear equation in x, it has either zero, one, or infinitely many solutions.  Obviously, we need the latter to occur, which happens when 9c=a^2 and 9b-9c-a^2 = 0, since the coefficient of x must cancel to zero and thus the RHS must equal zero as well.

Since 9c = a^2, we must have a = 3 \sqrt{c}.  Since a must be a multiple of three, we plug in values.  If a = 9, we get c = 9 and thus 9b - 162 = 0, which is impossible.  So a = 9 doesn't work.

With a = 6, c = 4, and thus 9b-72=0, so 9b=72, so b=8.  This gives 6+8+4=18 as one possibility.

With a = 3, c = 1, and thus 9b - 18 = 0, so b = 2.  This obviously is worse.

We've gone through all the cases and the two possibilities are 2 and 18, so our answer is \boxed{18}.


You might be interested in
4. Which graph shows a non-proportional linear<br> relationship between x and y?<br> A.<br> В.
Ghella [55]

Answer:

I think ( b) is non proportional linear

4 0
2 years ago
PLEASE HELP!!!
Klio2033 [76]

Answer:

The answer is A and B

Step-by-step explanation:

y-y=m(x-x)

y-(-3)=m(x-(-5))

y+3=0.4(x+5)

y-y=m(x-x)

y-1=0.4(x-5)

6 0
2 years ago
PLS HELPPPPPPPPPPPP WILL GIB BRAINLIEST Jimmy Needs to buy 1'000 boxs of Mac n' cheese for a confrecne meeting.
Montano1993 [528]

Answer:

its 500$ to buy 1000 mac and cheese at 50 cent a piece so im asuming the answer is A

Step-by-step explanation:

5 0
2 years ago
3, 8, 13, 18, 23, 28, 33, 38
algol [13]

The given series is

3,8,13,18,23,28,33,38,...

Where the first term, a , is 3.

And the difference is constant, that is

8-3=13-8=18-13=5

So the constant difference, d is 3.

Since the difference is constant, so the given series is arthmetic.

And for explicit rule, we use the formula of nth term, which is

a_{n}= a+(n-1)d

Substituting the values of a and d, we will get

a_{n} = 3+(n-1)5&#10;\\&#10;a_{n} = 3+5n-5&#10;\\&#10;a_{n} = 5n -2

Recursive rule is used to tell us the relationship between previous and current term. And the required rule is

a_{n} = a_{n-1} +5 , a_{1} = 3

3 0
3 years ago
0.2x + 0.3y = 1.3.<br>0.4r + 0.5y = 2.3. solve by using substitution ​
Alexxandr [17]

Answer:

x=2, y=3.

Step-by-step explanation:

0.2x + 0.3y = 1.3

0.4x + 0.5y = 2.3

From the first equation   y = (1.3 - 0.2x) / 0.3

Substituting in second equation:

0.4x + 0.5(1.3 - 0.2x) / 0.3 = 2.3

Multiply through by 0.3:

0.12x + 0.5(1.3 - 0.2x) =  0.69

0.12x + 0.65 - 0.1x = 0.69

0.02x = 0.04

x = 2.

Substituting for x in equation 1:

0.2* 2 + 0.3y = 1.3

0.4 + 0.3y = 1.3

0.3y = 1.3 - 0.4 = 0.9

y = 3.

Check in equation 2:

0.4 * 2 + 0.5(3) =   0.8 + 1.5 = 2.3.

- Correct.

4 0
2 years ago
Other questions:
  • What is the width of a rectangle with length 14 cm and area 161cm squared 2
    7·1 answer
  • 5.6 as an improper fraction simplified?
    10·1 answer
  • Luke’s dad need 165 for a new suitcase. If he saves an equal amount for 3 weeks, how much does save each week?
    11·1 answer
  • Divide 72 ft in the ratio 9:4:5
    5·2 answers
  • At Roosevelt High School, there are six more students on the
    7·1 answer
  • Find the equation of the line (in slope-intercept form) through the points (1,-4) and with slope 5/2.
    15·1 answer
  • Wat is the answer for this question -x2 + 5x + 24.
    14·1 answer
  • X^2-x-12, please show your work so I can know how to do it. Thank you:))
    15·1 answer
  • Which of the following conditions will guarantee that line l is parallel to line m in the diagram?
    8·1 answer
  • PLEASE ANSWER brainlest!
    12·2 answers
Add answer
Login
Not registered? Fast signup
Signup
Login Signup
Ask question!